Đến nội dung

Hình ảnh

Cho x, y, z >0 và x.y.z=1. Tìm GTNN của: $\frac{x^{3}+y^{3}+z^{3}}{x+y+z}$

bất đẳng thức cực trị

  • Please log in to reply
Chủ đề này có 5 trả lời

#1
ILoveMath4864

ILoveMath4864

    Trung sĩ

  • Thành viên
  • 122 Bài viết

1. Cho x, y, z >0 và x.y.z=1. Tìm GTNN của:

B=$\frac{x^{3}+y^{3}+z^{3}}{x+y+z}$

2. CHo a, b, c >0 thỏa mãn a+b+c=1. Chứng minh:

$\frac{ab+c}{c+1}+\frac{bc+a}{a+1}+\frac{ca+b}{b+1}\leq 1$

3. Tìm GTNN của biểu thức:

$E=\frac{a^{4}}{(b-1)^{3}}+\frac{b^{4}}{(a-1)^{3}}$

4. Cho các số dương a, b, c thỏa mãn điều kiện a+b+c=3. Hãy tìm GTNN của:

$\frac{b\sqrt{b}}{\sqrt{2a+b+c}}+\frac{c\sqrt{c}}{\sqrt{2b+c+a}}+\frac{a\sqrt{a}}{\sqrt{2c+a+b}}$


Bài viết đã được chỉnh sửa nội dung bởi ILoveMath4864: 11-03-2017 - 22:13


#2
NHoang1608

NHoang1608

    Sĩ quan

  • Thành viên
  • 375 Bài viết

4) Áp dụng bđt Cauchy-Schwarz thì ta có $(a\sqrt{a}+(b\sqrt{b}+(c\sqrt{c})(\sqrt{a}+\sqrt{b}+\sqrt{c})\geq (a+b+c)^{2}= 9$

Mà áp dụng bđt AM-GM thì ta có $\sqrt{a}+\sqrt{b}+\sqrt{c}\leq a+b+c=3$ suy ra $a\sqrt{a}+b\sqrt{b}+c\sqrt{c}\geq 3$ $(1)$

Lại áp dụng bđt AM-GM thì ta được $\sqrt{2a+b+c}\leq \frac{2a+b+c+4}{4}$ suy ra $\sum{sqrt{2a+b+c}}\leq \frac{1}{4}\sum{2a+b+c+4}=6$  $(2)$.

Sử dụng BĐT Cauchy-Schwarz thì ta có:

   $ (\sqrt{2a+b+c}+\sqrt{2b+c+a}+\sqrt{2c+a+b})(\frac{b\sqrt{b}}{\sqrt{2a+b+c}}+\frac{c\sqrt{c}}{\sqrt{2b+c+a}}\frac{a\sqrt{a}}{\sqrt{2c+b+a}}$

    $ geq (\sqrt{b\sqrt{b}}+

 

 

 

P/s: sai


Bài viết đã được chỉnh sửa nội dung bởi NHoang1608: 11-03-2017 - 23:01

The greatest danger for most of us is not that our aim is too high and we miss it, but that it is too low and we reach it.

----- Michelangelo----


#3
NHoang1608

NHoang1608

    Sĩ quan

  • Thành viên
  • 375 Bài viết

4) Ta có $P= \frac{b^{2}}{\sqrt{b}\sqrt{2a+b+c}}+\frac{c^{2}}{\sqrt{c}\sqrt{2b+c+a}}+\frac{a^{2}}{\sqrt{a}\sqrt{2c+b+a}}$

Ap dụng bđt $AM-GM$ thì ta được $\sqrt{4b}\sqrt{2a+b+c} \leq \frac{2a+5b+c}{2}$ suy ra $\sqrt{b}\sqrt{2a+b+c}\leq \frac{2a+5b+c}{8}$

Vì lẽ đó mà $P \geq 8(\frac{b^{2}}{2a+5b+c}+\frac{c^{2}}{2b+5c+a}+\frac{a^{2}}{2c+5b+a})$

Ap dụng bđt $Cauchy-Schwarz$ thì ta có $\frac{b^{2}}{2a+5b+c}+\frac{c^{2}}{2b+5c+a}+\frac{a^{2}}{2c+5b+a}\geq \frac{(a+b+c)^{2}}{8(a+b+c)}$

Suy ra $P\geq 8\frac{(a+b+c)^{2}}{8(a+b+c)}=a+b+c=3$

Đẳng thức xảy ra khi $a=b=c=1$.

 

 

P/s: Mai đăng tiếp. Buồn ngủ wa


Bài viết đã được chỉnh sửa nội dung bởi NHoang1608: 11-03-2017 - 23:14

The greatest danger for most of us is not that our aim is too high and we miss it, but that it is too low and we reach it.

----- Michelangelo----


#4
Kagome

Kagome

    Trung sĩ

  • Thành viên
  • 166 Bài viết

2. CHo a, b, c >0 thỏa mãn a+b+c=1. Chứng minh:

$\frac{ab+c}{c+1}+\frac{bc+a}{a+1}+\frac{ca+b}{b+1}\leq 1$

$\frac{ab+c}{c+1}=\frac{ab+c(a+b+c)}{(a+c)+(b+c)}\leqslant \frac{(a+c)(b+c)}{2\sqrt{(a+c)(b+c)}}=\frac{1}{2}\sqrt{(a+c)(b+c)}$.

Tương tự $\frac{bc+a}{a+1}\leqslant \frac{1}{2}\sqrt{(a+b)(a+c)},\frac{ca+b}{b+1}\leqslant \frac{1}{2}\sqrt{(c+b)(a+b)}$

$\sqrt{(a+c)(b+c)}+\sqrt{(a+b)(a+c)}+\sqrt{(c+b)(a+b)}\leqslant 2(a+b+c)$

$\Rightarrow BĐT\leqslant a+b+c=1$

Dấu $'='\Leftrightarrow a=b=c=\frac{1}{3}$.



#5
ILoveMath4864

ILoveMath4864

    Trung sĩ

  • Thành viên
  • 122 Bài viết

4) Ta có $P= \frac{b^{2}}{\sqrt{b}\sqrt{2a+b+c}}+\frac{c^{2}}{\sqrt{c}\sqrt{2b+c+a}}+\frac{a^{2}}{\sqrt{a}\sqrt{2c+b+a}}$

Ap dụng bđt $AM-GM$ thì ta được $\sqrt{4b}\sqrt{2a+b+c} \leq \frac{2a+5b+c}{2}$ suy ra $\sqrt{b}\sqrt{2a+b+c}\leq \frac{2a+5b+c}{8}$

Vì lẽ đó mà $P \geq 8(\frac{b^{2}}{2a+5b+c}+\frac{c^{2}}{2b+5c+a}+\frac{a^{2}}{2c+5b+a})$

Ap dụng bđt $Cauchy-Schwarz$ thì ta có $\frac{b^{2}}{2a+5b+c}+\frac{c^{2}}{2b+5c+a}+\frac{a^{2}}{2c+5b+a}\geq \frac{(a+b+c)^{2}}{8(a+b+c)}$

Suy ra $P\geq 8\frac{(a+b+c)^{2}}{8(a+b+c)}=a+b+c=3$

Đẳng thức xảy ra khi $a=b=c=1$.

 

 

P/s: Mai đăng tiếp. Buồn ngủ wa

chỗ đó phải là $\sqrt{b}\sqrt{2a+b+c}\leq \frac{2a+5b+c}{4}$ chứ bạn



#6
quangtohe

quangtohe

    Hạ sĩ

  • Thành viên
  • 88 Bài viết

Áp dụng bất đẳng thức Cheybeyshev ta có

$3(x^{3}+y^{3}+z^{3})\geq (x+y+z)(x^{2}+y^{2}+z^{2})\geq3(x+y+z)$

 

=> $B\geq 1$

Dấu = xảy ra <=> x=y=z=1


quangtohe1234567890






Được gắn nhãn với một hoặc nhiều trong số những từ khóa sau: bất đẳng thức, cực trị

0 người đang xem chủ đề

0 thành viên, 0 khách, 0 thành viên ẩn danh